You are on page 1of 52

DO NOT OPEN THIS TEST BOOKLET UNTIL YOU ARE TOLD TO DO SO

For registration
to this test series

TEST BOOKLET
GENERAL STUDIES
PAPER-I
D
Time Allowed : Two Hours Maximum Marks :200
INSTRUCTIONS
1. IMMEDIATELYAFTER THE COMMENCEMENT OFTHE EXAMINATION, YOUSHOULD CHECKTHAT THISTEST
BOOKLET DOES NOT HAVE ANY UNPRINTED OR TORN OR MISSING PAGES OR ITEMS, ETC. IF SO, GET IT
REPLACED BYACOMPLETETEST BOOKLET.
2. Please note that it is the candidate's responsibility to encode and fill in the Roll Number and Test Booklet Series Code
A, B, C or D carefully and without any omission or discrepancy at the appropriate places in the OMR Answer Sheet.
Any omission/discrepancy will render the Answer Sheet liable for rejection.
3. You have to enter your Roll Number on the
Test Booklet in the Box provided alongside.
DO NOT write anything else on the Test Booklet.
4. This Test Booklet contains 100 items (questions). Each item is printed both in Hindi and English. Each item comprises
four responses (answers). You will select the response which you want to mark on the Answer Sheet. In case you feel
that there is more than one correct response, mark the response which you consider the best. In any case, choose ONLY
ONE response for each item.
5. You have to mark all your responses ONLY on the separate Answer Sheet provided. See directions in the Answer Sheet.
6. All items carry equal marks.
7. Before you proceed to mark in the Answer Sheet the response to various items in the Test Booklet, you have to fill in
some particulars in the Answer Sheet as per instructions sent to you with your Admission Certificate.
8. After you have completed filling in all your responses on the Answer Sheet and the examination has concluded, you
should hand over to the Invigilator only the Answer Sheet. You are permitted to take away with you the Test Booklet.
9. Sheets for rough work are appended in the Test Booklet at the end.
10. Penalty for wrong answers:
THEREWILL BE PENALTYFORWRONGANSWERS MARKED BYACANDIDATE INTHE OBJECTIVETYPE
QUESTION PAPERS.
(i) There are four alternatives for the answer to every question. For each question for which a wrong answer has
been given by the candidate, one-third of the marks assigned to that question will be deducted as penalty.
(ii) If a candidate gives more than one answer, it will be treated as a wrong answer even if one of the given answers
happens to be correct and there will be same penalty as above to that question.
(iii) If a question is left blank, i.e., no answer is given by the candidate, there will be no penalty for that question.

Join our Telegram channel for Synopsis PDF... https://t.me/IAS21stcenturyHYD/121)


1) Consider the following statements regarding Which of the statements given above is/are not
Uniform Civil Code: correct?
1. It is a Directive Principle of State Policy a) 1 only
given under article 45 of Indian Constitution. b) 2 and 3 only
2. It is a Gandhian directive principle. c) 2, 3 and 4 only
3. Uniform Civil Code refers to the body of d) 1, 2 and 3
laws governing rights and duties pertaining
to property and personal matters like Answer : c
marriage, divorce, adoption and inheritance. Explanation :
Which of the above statements is/are correct? The act introduced a system of communal
a) 3 only representation for Muslims by accepting the
concept of separate electorate. Under this, the
b) 2 and 3 only Muslim members were to be elected only by
c) 1 and 2 only Muslim voters. Thus, the Act legalised
communalism .
d) 1, 2, and 3
Answer: a * Power to discuss budget was already accorded in
the Act of 1892.
Explanation:
* Bicameralism and direct elections were introduced
* A Uniform Civil Code means that all sections of by the act of 1919.
the society irrespective of their religion shall be
treated equally according to a national civil code, 3) Consider the following statements regarding
which shall be applicable to all uniformly. They black swan event:
cover areas like- Marriage, divorce, maintenance, 1) It is a rare and unpredictable event.
inheritance, adoption and succession of the 2) Covid-19 pandemic is an example of black
property.
swan event.
* It is a Directive Principle under the Constitution
Which of the above statements is/are correct?
(Article 44). It is a Liberal-intellectual principle.
a) 1 only
2) Consider the following statements with
reference to the Indian Councils Act of 1909: b) 2 only

1. It introduced a system of communal c) Both 1 and 2


representation in India. d) Neither 1 nor 2
2. It introduced direct elections in the country. Solution (a)
3. For the first time, the legislative councils * A study by the Reserve Bank of India (RBI)
got the power to discuss the budget. talked about the possibility of capital outflows of
4. It introduced bicameralism in the country.
KPIAS (2 - D) Cell: 91332 37733
$100 billion from India in case of a major global * To inquire into specific complaints with
risk scenario or a “black swan” event. respect to the deprivation of rights and
safeguards of the socially and educationally
* It is a rare, unpredictable event that comes as a
backward classes;
surprise and has a significant impact on society or
the world. * To participate and advise on the socio-
* Covid-19 pandemic is not considered as an economic development of the socially and
educationally backward classes and to
example of black swan event because it was
evaluate the progress of their development
predictable.
under the Union and any State;
4) It is a Constitutional body established under
* To present to the President, annually and at
the 102nd Constitutional Amendment Act.
such other times as the Commission may
Which body is it?
deem fit, reports upon the working of those
a) Goods and Service Tax Council safeguards;
b) National Commission for Backward Classes * To make in such reports the
c) Official Language Commission recommendations as to the measures that
should be taken by the Union or any State
d) National Commission for Scheduled Caste
for the effective implementation of those
Solution (b) safeguards and other measures for the
* Recently, the Union Cabinet gave the 13th protection, welfare and socio-economic
extension to the Justice Rohini Commission, giving development of the socially and
it time until January 31, 2023, to submit its report. educationally backward classes;

* National Commission for Backward Classes * To discharge such other functions in relation
(NCBC) is a constitutional body (123rd to the protection, welfare and development
constitutional amendment Bill 2017 and 102nd and advancement of the socially and
amendment 2018 in constitution to make it educationally backward classes as the
constitutional body) (Article 338B of the Indian President may, subject to the provisions of
Constitution) any law made by Parliament, by rule
specify.
* According to Article 338B, it shall be the duty
of the Commission: 5) The “Instrument of Instructions” which have
been incorporated in the Constitution of
* To investigate and monitor all matters India as Directive Principles of State Policy
relating to the safeguards provided for the were contained in the
socially and educationally backward classes
under the Constitution or under any other a) Government of India Act of 1935
law for the time being in force or under any b) Indian Independence Act of 1947
order of the Government and to evaluate
c) Government of India Act 1919
the working of such safeguards;
d) Nehru Report of 1928

KPIAS (3 - D) Cell: 91332 37733


Answer: a 7) Consider the following statements regarding
Explanation: ‘the Charter Act of 1813’:

The Directive Principles of State Policy resemble 1. The company was allowed to continue
the ‘Instrument of Instructions ‘enumerated in the territorial possession for period of 20 years
Government of India Act of 1935. Those 2. It ended the trade monopoly of the East
instructions were issued to the Governor General India Company in India except for trade in
and Governors of colonies of India by the British tea and trade with China.
Government.
3. It asserted the sovereignty of the British
Crown over the Indian territories held by
6) Consider the following statements regarding the Company.
the International Conference for Election 4. The Charter Act of 1813 required the
Management Bodies (EMBs), 2022: Company to maintain its territorial and
1) It was hosted by the Ministry of Statistics commercial accounts separately.
and Programme Implementation (MOSPI). Select the correct answer using the code given
2) It was held under the aegis of the United below.
States’ ‘Summit for Democracy’ platform. a) Only one of the statements is correct.
Which of the statements given above is/are b) Only two of the statements are correct.
correct? c) Only three of the statements are correct.
a) 1 only
d) All four statements are correct.
b) 2 only
Answer : d
c) Both 1 and 2 Explanation :
d) Neither 1 nor 2 The Charter Act, 1813 :
Solution (b) * Monopoly of East India Company to trade with
* The Chief Election Commissioner addressed an India was brought to an end but the company
international conference for EMBs hosted by the retained the trade with China and the trade in tea.
Election Commission of India (ECI). * The company’s shareholders were given a 10.5
* The conference was held under the aegis of the percent dividend on the revenue in India.
United States’ ‘Summit for Democracy’ platform. * The company was allowed to continued territorial
* ‘Summit for Democracy’, was an initiative of the possession for period of 20 years i.e. from 1833
US President hosted in December 2021. It brings to 1853, without prejudice to the sovereignty of
together government, civil society, and the private the Crown. The company was held in trust to the
sector leaders in a shared effort. Crown.
* The Charter Act of 1813 required the Company

KPIAS (4 - D) Cell: 91332 37733


to maintain its territorial and commercial accounts socio-economic development and other welfare
separately. activities of the SCs.
* By this Act the Company was put under greater * Any other function with respect to the welfare,
control of the British Crown, but could retain its protection, development and advancement of the
hold on the details of administration and Indian SC community.
revenues. From 1853, the revenues of India were 9) Which of the following Fundamental Rights
controlled by the British Parliament.
are protected against only State action and
8) Which of the following is not the function of not against actions taken by private
the National Commission for Scheduled individuals?
Castes? 1. Right to freedom of speech and expression
a) Providing Constitutional protection to
2. Right to prohibition of discrimination on
Scheduled Castes.
grounds of religion, race, caste, sex or place
b) To investigate any case that violates the of birth.
interests of Scheduled Castes. 3. Right to Equality before law and equal
c) To submit the report to the Prime Minister protection of laws
related to the protection of Scheduled 4. Right to life and liberty
Castes.
Select the correct answers using the codes
d) Recommending steps to be taken to further
given below
the socio-economic development and other
welfare activities of the SCs. a) 3 and 4 only

Solution (c) b) 1, 2 and 3 only

Functions of National Commission for Scheduled c) 3 only


Castes: d) 1, 2 and 4 only
* Monitoring and investigating all issues concerning Answer: c
the safeguards provided for the SCs under the
Explanation:
constitution.
* Generally, the fundamental rights are enforceable
* Enquiring into complaints relating to the deprivation
against the state whereas in few cases can be
of the rights and safeguards of the SCs.
enforceable against the private individuals.
* Taking part in and advising the central or state
* The following fundamental rights can be enforced
governments with respect to the planning of socio-
against action taken by private individuals.
economic development of the SCs.
* Article 15(2): Obliges all public and private
* Regular reporting to the President of the country
entities to abstain from doing discrimination on
on the implementation of these safeguards.
ground of sex, caste, race or place of birth.
* Recommending steps to be taken to further the
* Article 17: Abolition of untouchability
KPIAS (5 - D) Cell: 91332 37733
* Article 23: Prohibits human trafficking and forced 11) Which among the following was/were the
labour reasons behind making Directive Principles
non-justiciable and legally non enforceable by
* Article 24: Prohibits employment of children in
factories and hazardous places the makers of the Constitution?
1. Lack of financial resources in the country
* In January 2023, Kaushal Kishor v. State of
U.P., Supreme Court by 4:1 majority held that to implement them.
Article 19 & 21 can be enforced against 2. Implementation challenge due to vast
private individuals & entities. diversity and backwardness in the country.
* The court took this view while ruling that the right 3. Making them justiciable would widely
of free speech and expression guaranteed under increase the power of courts and judiciary.
the Article 19(1)(a) cannot be curbed by any
4. The newly independent India should be free
additional grounds other than those already laid
to decide, the order, the time, the place and
down in Article 19(2). the mode of fulfilling them.
Note: Habeas corpus (Article 32) is maintainable Select the correct answer using the codes
against private individuals and private bodies also.
given below.
10) A report titled ‘A proposal for Digital Banks a) 1 and 2 only
in India: Licensing and Regulatory Regime’
was recently released by which of the b) 1, 2, 3 and 4
following? c) 1, 2 and 3 only
a) Reserve Bank of India d) 1, 2 and 4
b) NITI Aayog Answer: d
c) National Payments Corporation of India Explanation: Statement 3 is INCORRECT
(NPCI)
* The framers of the Constitution made the Directive
d) Ministry of Electronics & Information Principles non-justiciable and legally non
Technology (MeitY) enforceable because:
Solution (b) a) The country did not possess sufficient
* Recently, the Government think tank NITI Aayog financial resources to implement them.
released its report titled ‘A proposal for Digital b) The presence of vast diversity and
Banks in India: Licensing and Regulatory Regime’. backwardness in the country would stand
* The report focuses on avoiding any regulatory or in the way of their implementation.
policy arbitrage and offers a level playing field to c) The newly born independent Indian State
incumbents as well as competitors. It offers a with its many preoccupations might be
template and roadmap for licensing and regulatory crushed under the burden unless it was free
regime for India. to decide the order, the time, the place and
the mode of fulfilling them.
KPIAS (6 - D) Cell: 91332 37733
12) With reference to the Election Commission convention and several judicial pronouncements,
of India (ECI), consider the following once the actual process of elections has started,
statements: the judiciary does not intervene in the actual
conduct of the polls. Once the polls are completed
1) The Election Commissioners enjoy the
and results declared, the Commission cannot
same status as available to the Chief Justice
review any result on its own. This can only be
of the High Court.
reviewed through the process of an election petition,
2) The Chief Election Commissioner can be which can be filed before the High Court, in
removed from office only through respect of elections to the Parliament and State
impeachment by Parliament. Legislatures. In respect of elections for the offices
3) The decisions of the ECI can be challenged of the President and Vice President, such petitions
only in the Supreme Court of India by can only be filed before the Supreme Court.
appropriate petitions.
Which of the statements given above is/are 13) The protection against double jeopardy is
correct? available only in proceedings before an
a) 1 and 2 only authority of a judicial nature. In this context,
the term “judicial authority” means:
b) 2 only
a) The constitutional authority vested in courts
c) 2 and 3 only
to hear and decide justiciable cases.
d) 1, 2 and 3
b) The authority to interpret statutes when
Solution (b) disputes arise over their constitutionality.
* Recently, the new Chief Election Commissioner c) The authority concerned with
(CEC), Rajiv Kumar has taken charge of the implementation of government policies and
Election Commission of India (ECI). enforcement of duly enacted laws.
* The President appoints Chief Election d) Both (a) and (b)
Commissioner and Election Commissioners. They
Answer: d
have tenure of six years, or up to the age of 65
years, whichever is earlier. They enjoy the same * Explanation: Article 20 provides the protection
status and receive salary and perks as available against double jeopardy i.e., no person shall be
to Judges of the Supreme Court of India. prosecuted and punished for the same offense
more than once.
* The Chief Election Commissioner can be removed
from office only through impeachment by * Judicial authority means court of law or a judicial
Parliament. tribunal. The departmental or administrative
authorities are not included under this.
* The decisions of the Commission can be challenged
in the High Court and the Supreme Court of India Judicial authority includes
by appropriate petitions. By long standing i. The constitutional authority vested in courts

KPIAS (7 - D) Cell: 91332 37733


to hear and decide justiciable cases. general superintendence over vigilance
ii. The authority to interpret statutes when administration.
disputes arise over their constitutionality. * The Commission was given statutory status by an
enactment of the CVC Act, 2003 and vested with
14) Which of the following commissions is a
autonomy and insulation from external influences.
Constitutional body?
a) National Commission for Women * After the enactment of the CVC Act, 2003, the
Commission became a multi-member body
b) National Commission for Scheduled Caste consisting of a Central Vigilance Commissioner
c) National Commission for Minorities (Chairperson) and not more than two Vigilance
Commissioners (Members).
d) National Commission for Human Rights
Solution (b)
16) Consider the following statements regarding
the Parliamentary form of Government in
15) Consider the following statements regarding India?
Central Vigilance Commission:
1. Watertight separation of powers between
1) It has its origin in the Special Police three organs of the government.
Establishment set up in 1941.
2. Executive is responsible to the legislature.
2) The Commission was given statutory status
3. Division of powers between central
by an enactment of the CVC Act, 2003.
government and state government.
3) The Commission consists of a Central
4. Concept of Nominal and Real Executives.
Vigilance Commissioner and not more than
ten Vigilance Commissioners. Select the correct answer using the code
given below.
Which of the statements given above is/are
correct? a) Only one of the statements is correct.
a) 1 only b) Only two of the statements are correct.
b) 2 only c) Only three of the statements are correct.
c) 2 and 3 only d) All four statements are correct.

d) 1, 2 and 3 Answer : b
Solution (b) Explanation :
* Central Vigilance Commission was set up by the * In the parliamentary form of the government there
Government of India (Ministry of Home Affairs) will not be watertight separation of the powers.
vide Resolution in 1964 on the recommendation Here the executive is part of the legislature.
of the Santhanam Committee. It is an apex body * Here, executive is answerable to the legislature,
for the prevention of corruption and exercising unlike presidential form of the government, where
KPIAS (8 - D) Cell: 91332 37733
executive is not answerable to the legislature. and the guardian of the
* Division of powers refers to decentralization * fundamental rights. Also, judiciary cannot
and autonomy to the states rather than get into law making and only acts as a
parliamentary form of government. corrective force when the other two pillar:
executive and legislative fail in their
* Also, President is the Nominal Executive and
functions.
Prime Minister is the real executive in
Parliamentary form of Government. * Judiciary in the interpreter of the constitution
Hence, Statement 2 and Statement 4 are and the guardian of the fundamental rights.
Also, judiciary cannot get into law making
correct.
and only acts as a corrective force when
the other two pillar: executive and legislative
17) With reference to the Constitution of India, fail in their functions.
the Fundamental Rights constitute limitations
upon
18) Which the following statement(s) regarding
1. Legislative function. minorities is/are correct with respect to the
2. Administrative function Indian Constitution?
3. Judicial function 1. The Constitution grants protection to both
religious and linguistic minorities.
4. Executive function
2. The Constitution grants the right of
Which of the above statements is/are correct?
establishing and administering educational
a) 1 and 3 only institutions for minorities.
b) 4 only 3. The term Minority has nowhere been
c) 1, 2 and 4 only defined in the Constitution.

d) All the above Select the correct answer using the code
given below
Answer: c
a) 1 and 3 only
Explanation: Statement 3 is INCORRECT
b) 2 and 3 only
* The Fundamental Rights place limitations
upon the arbitrary laws of the legislative. c) 1 and 2 only
Recent privacy verdict based on the Aadhar d) 1, 2 and 3
bill is an example. It is also available against
Answer: d
administrative actions. For e.g.: quo-
warranto and certiorari which are writs Explanation:
defined under Article 32 are available * The Right of Minorities to Establish and Administer
against the administrative actions of a body Educational Institutions (Article 30) grants rights
Judiciaryin the interpreter of the constitution to minorities, whether religious or linguistic.
KPIAS (9 - D) Cell: 91332 37733
* In granting aid, the State shall not discriminate Answer: b
against any educational institution managed by a Explanation: According to Article 12, the State includes
minority. Thus, the protection under Article 30 is
the following:
confined only to minorities (religious or linguistic)
and does not extend to any section of citizens (as (a) Government and Parliament of India, that
under Article 29). is, executive and legislative organs of the
Union government.
* However, the term ‘minority’has not been defined
anywhere in the Constitution. The right under (b) Government and legislature of states, that
Article 30 also includes the right of a minority to is, executive and legislative organs of state
impart education to its children in its own language. government.

Note: Muslims, Sikhs, Christians, Buddhists, Jain and (c) All local authorities that is, municipalities,
Zoroastrians (Parsis) have been notified as panchayats, district boards, improvement
minority communities under Section 2 (c) of the trusts, etc.
National Commission for Minorities Act, 1992. (d) All other authorities, that is, statutory or non-
statutory authorities like LIC, ONGC, SAIL,
etc.
19) The term ‘State’ has been used in different
provisions concerning the fundamental * Thus, State has been defined in a wider sense so
rights. According to Article 12, the State as to include all its agencies. It is the actions of
includes which among the following entities these agencies that can be challenged in the courts
as violating the Fundamental Rights. According to
1. Executive organs of the Union and state the Supreme Court, even a private body or an
governments. agency working as an instrument of the State falls
2. Legislative organs of the Union and state within the meaning of the ‘State’ under Article
governments. 12.
3. Judiciary at Union and state levels.
4. Local Self-governments created under 73rd 20) Which Article of the Constitution places a
and 74th amendments. duty upon the State to provide a lawyer to
5. Statutory authorities. any citizen who is unable to engage one due
to poverty and other disability?
6. Non-statutory authorities.
a) Article 38
Choose the correct codes
b) Article 39 A
a) 1, 2, 3, 4 and 5 only
c) Article 39 (b)
b) 1, 2, 4, 5 and 6 only
d) Article 39 (c)
c) 1, 2, 3 and 4 only
Answer: b
d) All the above
Explanation:

KPIAS (10 - D) Cell: 91332 37733


* Article 39A imposes an obligation on the State to the president on the recommendations of a six-
provide free legal aid to ensure access to justice member committee consisting of the prime minister
for all citizens. Legal aid is defined as “such legal as its head, the Speaker of the Lok Sabha, the
aid” as may be provided by the state under the Deputy Chairman of the Rajya Sabha, leaders of
Legal Services Authorities Act, 1987 or any other the Opposition in both the Houses of Parliament
law for the time being in force. The right to free and the Central home minister.
legal aid is thus limited only by the extent of funds
made available in any particular year by the
legislature. 22) ‘Operation Garuda’ recently seen in the news
relates to which of the following?
a) Evacuation of Indian citizens from Yemen
21) Consider the following statement regarding
the NHRC: b) Combined operation against Naxalites

1) NHRC is a statutory body established under c) Against illicit drug trafficking network
the National Human Rights Commission d) Rescue, and relief operation in flood
Act, 2006.
Solution (c)
2) The chairman and members are appointed
* The Central Bureau of Investigation has
by the President on the recommendations
launched a multi-phase ‘Operation Garuda’ against
of a six-member committee consisting of
illicit drug trafficking network, registering 127 new
the Prime Minister as its head.
cases, arresting 175 people and seizing huge
Which of the statements given above is/are quantities of narcotic drugs.
correct?
* It was initiated in close coordination with Interpol
a) 1 only and Narcotics Control Bureau, for combating the
b) 2 only smuggling of illicit drugs and psychotropic
substances, with special focus on the Indian Ocean
c) Both 1 and 2 region.
d) Neither 1 nor 2
Solution (b)
23) Consider the following statements regarding
* The National Human Rights Commission is a the meaning of the State and Law which have
statutory (and not a constitutional) body. It was been mentioned in Part III of the
established in 1993 under a legislation enacted by Constitution:
the Parliament, namely, the Protection of Human 1. Article 12 of the Constitution provides the
that is, the rights relating to life, liberty, equality
definition of State.
and dignity of the individual guaranteed by the
Constitution or embodied in the international 2. Article 13 provides for the doctrine of
covenants and enforceable by courts in India. judicial review and the definition of law.

* The chairperson and members are appointed by


KPIAS (11 - D) Cell: 91332 37733
3. AConstitutional Amendment is included in c) Both 1 and 2
the meaning of law as given under Article d) Neither 1 nor 2
13.
Answer: a
Which of the statements given above is/are
correct? Explanation: European Commission, in a global first,
proposed a set of digital rights and principles
a) 1 only recently (January 2022). It aims to protect people’s
b) 2 and 3 only rights, support democracy and ensure a fair and
c) 1 and 2 only safe online environment.

d) 1, 2 and 3 * Digital rights, closely linked to freedom of


expression and privacy, are those that allow people
Answer: c to access, use, create and publish digital media, as
Explanation: + well as access and use computers, other electronic
devices and communications networks.
* The Supreme Court in Shankari Prasad vs. Union
of India 1951 case held that the term “law” as * Digital rights are merely an extension of the rights
given under Article 13(2) includes ordinary set out in the Universal Declaration of Human
legislations but not Constitutional Amendments. Rights by the United Nations as applied to the
Article 13(3) that defines what is “law”, does not online world.
include a Constitutional Amendment Act under it. * It is a broad term implying right to privacy and
* Further, Article 13(4), which was added by the data protection; it can be related to trolling, online
24th Constitutional Amendment Act 1971, threats and hate speech; it can address broader
excludes Constitutional Amendments from the issues of equitable Internet access regardless of
purview of the definition of “law’ as used under economic backgrounds and disabilities.
Article 13(2).

25) Which among the following provisions are


24) With reference to Digital Rights, consider the mentioned in Part IV of Indian Constitution?
following statements. 1. Reserving jobs for scheduled caste and tribe
1. They are linked to freedom of expression in government
and privacy. 2. State to promote the use of Hindi
2. “The Declaration on Digital Rights and 3. Impart primary education to linguistic
Principles”, was recently signed by the minorities in their mother tongue.
United Nations.
4. Separation of judiciary from executive
Select the correct answer using the code
given below. Select the correct answer using the code
given below.
a) 1 only
b) 2 only
KPIAS (12 - D) Cell: 91332 37733
a) 2 and 4 only 2. Representation of People Act 1951.
b) 4 only 3. The Undisclosed Foreign Income and
c) 1,2, 3 and 4 Assets (Imposition of Tax) Bill, 2015.
Choose the correct codes
d) 1,2 and 3 only
a) 1 and 2 only
Answer: b
Explanation: ONLY Statement 4 is CORRECT b) 1 and 3 only

* Article 50 - To separate the judiciary from the c) 2 and 3 only


executive in the public services of the State. d) All the above
Directives outside Part-IV Directive Principles of Answer: a
State Policy Explanation:
* Article 335, Part XVI - The claims of scheduled
* The Verma Committee on Fundamental Duties of
cast and scheduled tribes shall be taken into
the Citizens (1999) identified the existence of legal
consideration, consistently with the maintenance
provisions for the implementation of some of the
of efficiency of administration, in the making of Fundamental Duties.
appointments to services and posts in connection
with the affairs of the Union or a State. * The Unlawful Activities (Prevention) Act of 1967
provides for the declaration of a communal
* Article 350 A, XVII - It shall be the duty of every organisation as an unlawful association. Hence,
state and every local authority within the state to statement 1 is correct.
provide adequate facilities for instruction in the
mother tongue at the primary stage of education * The Representation of People Act (1951) provides
to Children belonging to the linguistic minority for the disqualification of members of the
groups. Parliament or a state legislature for indulging in
corrupt practice that is, soliciting votes on the
* Article 351, Part XVII - It shall be the duty of ground of religion or promoting enmity between
the Union to promote the spread of the Hindi different sections of people on grounds of caste,
language and to develop it so that it may serve as race, language, religion and so on. So, statement 2
a medium of expression for all the elements of
is also correct.
composite culture of India.
* Duty to pay taxes is not considered as
Fundamental Duty. Hence, statement 3 is wrong.
26) Which among the following legal provisions
are meant for the implementation of some of
the Fundamental Duties enshrined in the 27) With reference to the NITI Aayog, consider
constitution of India? the following statements:

1. Unlawful activities (Prevention) Act 1967. 1) It is an executive body setup in 2013 to


replace the Planning Commission.

KPIAS (13 - D) Cell: 91332 37733


2) The Chief Executive Officer appointed by nominated by the Prime Minister.
the Prime Minister is the chairperson of Full-time organizational framework:
NITI Aayog.
* Prime Minister as the Chairperson
Which of the statements given above is/are
correct? * Vice-Chairperson (appointed by the Prime
Minister)
a) 1 only
Members:
b) 2 only
* Full-time
c) Both 1 and 2
* Part-time members on a rotational basis: Maximum
d) Neither 1 nor 2 of 2 members from foremost universities, leading
Solution (d) research organizations, and other innovative
organizations in an ex-officio capacity.
* It was set up in 2015 to replace Planning
Commission. It is an Executive Body (acts as a * Ex Officio members: Maximum of 4 members of
think tank and advisory body). the Council of Ministers which is to be nominated
* Its objective is to foster the spirit of Cooperative by the Prime Minister.
and Competitive federalism through structured
support initiatives on a continuous basis. 28) Consider the following statements regarding
Composition of NITI Aayog: the ‘Competitive Federalism’:
* Chairperson: Prime Minister of India. 1. It involves competition among the States
* Governing Council consists of the Chief Ministers horizontally.
of all the States and Lt. Governors of Union 2. It will not involve vertical competition
Territories in India. The NITI Aayog’s between the Central government and State
governing council is the premier body tasked with governments.
evolving a shared vision of national priorities Which of the statements given above is/are
and strategies with the involvement of the
correct?
States and Union Territories. It presents a
platform to discuss inter- sectoral, inter- a) 1 only
departmental and federal issues. b) 2 only
* Regional Councils will be created to address c) Both 1 and 2
particular issues and possibilities affecting more
d) Neither 1 nor 2
than one state. These will be formed for a fixed
term and summoned by the PM. These will be Answer : a
chaired by the Chairperson of the NITI Aayog or Explanation :
his nominee.
* Competitive federalism refer to the relationship
* Special invitees: Eminent experts, specialists between the Central and state governments
with relevant domain knowledge, who will be
KPIAS (14 - D) Cell: 91332 37733
(vertical) or between state governments court for the enforcement of the fundamental rights
(horizontal). In a free-market economy, the during a national emergency (Article 359).
endowments of states, available resource base and
* Article 32(3), empowers Parliament to authorize
their comparative advantages all foster a spirit of
any other court to issue these writs. The Supreme
competition. States need to compete among
Court held that Article 32 is an important and
themselves and also with the Centre for benefits.
integral part of the basic structure of the
Constitution.
29) With reference to Article 32, which of the
following statements is incorrect? 30) Consider the following statements :
a) It is the basic feature of the Indian 1. Parliamentary form of Government enables
Constitution. cooperation between the legislature and the
b) The Right to move to the Supreme Court executive while ensuring complete power
can be suspended as provided in the separation.
Constitution. 2. Parliamentary form of Government ensures
c) The President of India can empower any collective as well as individual responsibility
court to issue writs. of the Executive.
d) Supreme Court can issue writs for 3. Parliamentary form of Government
enforcement of any Fundamental Rights. generally ensures political homogeneity of
Answer: c the Executive.

Explanation: Select the correct answer using the code given


below.
* Article 32 of the Constitution (Right to
Constitutional Remedies): It is a fundamental right, a) None of the statements is correct.
which states that individuals have the right to b) Only one of the statements is correct.
approach the Supreme Court (SC) seeking c) Only two of the statements are correct.
enforcement of other fundamental rights
recognised by the Constitution. d) All three of the statements are correct.

* The SC has power to issue directions or orders or Answer : c


writs for the enforcement of any of the Explanation :
fundamental rights. The writs issued may include
* Every modern government has three organs, the
habeas corpus, mandamus, prohibition, certiorari
Legislature, the Executive and the Judiciary. On
and quo-warranto.
the basis of the relationship between the executive
* The right to move the SC shall not be suspended and the legislature, democratic governments are
except as otherwise provided for by the classified into :
Constitution. Thus, the Constitution provides that
1. Parliamentary Form of government in which the
the President can suspend the right to move any

KPIAS (15 - D) Cell: 91332 37733


executive is selected from among the members 2. It refers to a situation where civil
of the legislature and is responsible to the administration is run by military authorities
legislature. However, it violates the theory of according to their own rules and regulations.
sepa7ration of powers and leads to the
3. The expression ‘Martial Law’ has not been
concentration of legislative and executive powers
defined anywhere in the constitution.
in the hands of the cabinet.
Select the correct code from the following:
The Cabinet is collectively responsible and
Each Minister is individually responsible to the a) 1 and 2
legislature. It means that once a decision is taken b) 2 and 3
by the Cabinet, it becomes the responsibility of
c) 1 and 3
each minister to support it, in and outside the
parliament. d) All of the above
2. Presidential Form of government in which the Answer: d
executive is completely separated from the Explanation:
legislature. The members of executive are not
the members of the legislature. The executive is * Article 34 provides for the restrictions on FRs
not accountable to the legislature. President is while martial law is in force in any area within the
head of state as well as head of government territory of India. Parliament is authorized to
indemnify any government servant for restoration
Political homogeneity means that the of law and order whilst martial law is in force even
members of the cabinet usually belong to the same if those actions were violative of Fundamental
political party. Usually they are taken from the Rights.
majority party in the legislature. But there are
instances when no single party secures the * The concept of Martial law has been borrowed
majority in the legislature. In that case, the only from British Common Law.
alternative is the formation of a coalition * Despite its mention, the term ‘martial law’ has not
government, consisting of more than one political been defined in the constitution and there are no
parties. grounds mentioned for the imposition of Martial
Hence, Statements 2 and 3 are correct. law.
* During the operation of martial law, the military
authorities are vested with abnormal powers to
31) Article 34 of the Indian Constitution provides take all necessary steps. However, the SC held
for the restriction on fundamental rights while Habeas Corpus is not suspended when martial law
‘Martial Law’ is in force in any area within is in force.
the territory of India. Which of the following
statements regarding ‘Martial Law’ are
correct? 32) Consider the following statements:
1. The concept of Martial law has been 1) At present, non-resident Indians (NRI)
borrowed from the ‘English Common Law’. settled in foreign land cannot become an
KPIAS (16 - D) Cell: 91332 37733
elector in the electoral roll in India. 33) Consider the following statements
2) A person who is not a citizen of India is not 1) The Election Commission of India (ECI)
eligible for registration as a voter in the does not have the power to deregister
electoral rolls in India. parties that are inactive.
Which of the statements given above is/are 2) A party can be de-registered if it is declared
correct? illegal by the Central Government.
a) 1 only Which of the statements given above is/are
b) 2 only correct?

c) Both 1 and 2 a) 1 only

d) Neither 1 nor 2 b) 2 only


c) Both 1 and 2
Solution (b)
* Recently, the Union government told the Supreme d) Neither 1 nor 2
Court (SC) that it was considering ways to Solution (c)
facilitate non-resident Indians (NRI), especially * Recently, the Election Commission ordered the
migrant labourers, to cast their votes remotely. delisting of 86 registered unrecognised political
* An overseas elector is a person who is a citizen parties it found to be “non-existent” and declared
of India and who has not acquired citizenship of 253 others “inactive”.
any other country and is otherwise eligible to be * The Election Commission of India (ECI) is not
registered as a voter and who is absenting from empowered to de-register parties on the grounds
his place of ordinary residence in India owing to of violating the Constitution or breaching the
his employment, education or otherwise is eligible undertaking given to it at the time of registration.
to be registered as a voter in the constituency in ECI has the power to register parties under the
which his place of residence in India as mentioned Representation of the People Act, (RPA) 1951,
in his passport is located. but it does not have the power to deregister parties
* According to the provisions of Section 20A of the that are inactive.
Representation of People Act, 1950, an NRI * A party can only be de-registered if its registration
settled in foreign land can become an elector in was obtained by fraud; if it is declared illegal by
the electoral roll in India. the Central Government; or if a party amends its
* A person who is not a citizen of India is not eligible internal Constitution and notifies the ECI that it
for registration as a voter in the electoral rolls in can no longer abide by the Indian Constitution.
India. Even those who have ceased to be citizens
of India on acquiring the citizenship of another
country are not eligible to be enrolled in the 34) Consider the following statements :
electoral rolls in India. 1. According to the Constitution of India,
States have no right to secede from the

KPIAS (17 - D) Cell: 91332 37733


federation. a) 1 and 2 only
2. Indian federation is the result of an b) 1 only
agreement among the states. c) 1 and 3 only
Which of the statements given above is/are
d) 3 only
correct?
Solution (d)
a) 1 only
* Article 165 states that the Governor of each State
b) 2 only shall appoint a person who is qualified to be
c) Both 1 and 2 appointed a Judge of a High Court to be Advocate-
General for the State.
d) Neither 1 nor 2
* When considering the qualification of a person for
Answer : a
appointment as an Advocate-General, the question
Explanation : is only whether he is qualified to be appointed as a
* In Indian federation, states have no right to secede Judge of the High Court as laid down in article
from the federation. Indian federation is not the 217 (2), and not whether he has attained the age
result of an agreement among the states like the at which he would have to retire had he been
American federation and the states have no right appointed a Judge of the High Court.
to secede from the federation. * In the performance of his official duties, the
* The federation is an Union and it is indestructible. advocate general is entitled to appear before any
Hence, the country is an integral whole and divided court of law within the state.
into different states only for the convenience of * The term of office of the advocate general is not
administration. fixed by the Constitution. He holds office during
Hence, only statement 1 is correct. the pleasure of the Governor.
* Conventionally, he resigns when the government
(council of ministers) resigns or is replaced, as he
35) Which of the following statements is/are
is appointed on its advice.
correct regarding the Advocate-General for
a State? * He may also quit his office by submitting his
resignation to the Governor.
1) He is constitutionally obligated to resign
when the council of ministers resigns.
2) He cannot appear in a court lower in rank 36) With reference to Prompt Corrective Action
to a district court. Framework (PCAF), consider the following
statements:
3) He may also quit his office by submitting
his resignation to the governor. 1) Reserve Bank of India has put in place a
PCA Framework for Scheduled
Select the correct answer using the code
Commercial Banks as well as Non-Banking
given below:
KPIAS (18 - D) Cell: 91332 37733
Financial Company (NBFCs). of the following corrective actions may be
2) When a bank is placed under PCA, prescribed:
restrictions on branch expansion, both * Restriction on dividend distribution/
domestic and overseas, may be prescribed. remittance of profits.
Which of the statements given above is/are * Promoters/shareholders to infuse equity and
correct? reduction in leverage; Restriction on issue
a) 1 only of guarantees or taking on other contingent
liabilities on behalf of group companies (only
b) 2 only for Core Investment Companies).
c) Both 1 and 2 * Restriction on branch expansion; domestic
d) Neither 1 nor 2 and/or overseas.
Solution (c) * Appropriate restrictions on capital
expenditure, other than for technological
* Recently, the Reserve Bank of India (RBI)
upgradation within Board approved limits.
removed the Central Bank of India from its Prompt
Corrective Action Framework (PCAF). * Appropriate restrictions on capital
expenditure, other than for technological
* Reserve Bank of India had introduced a Prompt
upgradation within Board approved limits.
Corrective Action Framework (PCA) for
Scheduled Commercial Banks in 2002 and the * Restrictions/reduction in variable operating
same has been reviewed from time to time based costs.
on the experience gained and developments in the
banking system.
37) Consider the following statements :
* The objective of the PCA Framework is to enable
1. Indian Councils Act 1909, introduced non-
Supervisory intervention at appropriate time and
require the Supervised Entity to initiate and official majority in all the Provincial
implement remedial measures in a timely manner, Legislative Councils for the first time.
so as to restore its financial health. The PCA 2. The Indian Councils Act 1909 also for the
Framework is also intended to act as a tool for first time gave members of the Council
effective market discipline. power to move resolutions on any matter
* It has now been decided to put in place a PCA of general public interest.
Framework for Non-Banking Financial Company Which of the statements given above is/are
(NBFCs) to further strengthen the supervisory correct?
tools applicable to NBFCs. It shall be applicable
a) 1 only
with effect from October 01, 2022 based on the
financial position of NBFCs on or after March 31, b) 2 only
2022. c) Both 1 and 2
* When a bank is placed under PCA, one or more d) Neither 1 nor 2
KPIAS (19 - D) Cell: 91332 37733
Answer : c c) Kesavananda Bharati v/s State of Kerala
Explanation : d) Maneka Gandhi v. Union of India
* The Indian Councils Act 1909 was introduced by Answer: b
the British government in India as a step towards
Explanation:
including Indians in government. It is also referred
to as the Morley-Minto reforms * In the case of A.K Gopalan vs the State of Madras,
the Supreme Court denied recognising the
* Its core feature was the recognition of the deficiencies of procedure established by law and
principle of elections of members to the central due process of law was rejected by the court. The
and provincial legislative councils. court clarified that ‘procedure established by law’
- The articles of the Act did the following: and ‘due process of law,’ i.e., the principles of
- increased the size of various provincial natural justice are different from each other.
legislative councils, However, this was reversed in the case of Maneka
Gandhi vs Union of India.
- created executive councils in the provinces
of Bombay, Madras and West-Bengal, and
- introduced the office of a vice-president‘ 39) Which among the following provisions of the
at both at the centre and the provinces Constitution shows that the Indian federation
is ‘federal in form but, unitary in spirit’?
- It created non-official majority in all the
Provincial Legislative Councils for the first 1. Single Constitution
time but maintained an official majority in 2. Provision of All India Services
the Central Legislative Council. 3. Appointment of State Governor by the
- It also for the first time gave members of Central government
the Council power to move resolutions on 4. Division of powers.
any matter of general public interest and to
divide the Council upon them. 5. Bicameralism.
6. Supremacy of the constitution.

38) In which of the following cases, did the Select the correct answer using the code given
majority of the Supreme Court propound the below.
view that by adopting the expression a) 1,3, 4, 5 and 6 only
‘Procedure Established by Law’, Article 21
b) 2, 3 and 6 only
of our Constitution had embodied the English
concept of personal liberty in preference to c) 1, 2 and 3 only
the American ‘Due Process’? d) 1, 2 3, 4, 5 and 6
a) Golaknath v. State of Punjab Answer : c
b) A.K. Gopalan v. State of Madras Explanation :

KPIAS (20 - D) Cell: 91332 37733


The Constitution of India establishes a federal its origin in the Special Police Establishment set
system of Government. It contains all the usual up in 1941 to probe bribery and corruption during
features of a federation, viz., two Government, World War II. It was set up by a resolution of the
division of powers, written Constitution, supremacy Ministry of Home Affairs in 1963 after the
of Constitution, rigidity of Constitution, independent Santhanam committee recommendation.
judiciary and bicameralism. * It functions under the Department of Personnel,
* However, the Indian Constitution also contains a Ministry of Personnel, Pension & Public
large number of unitary or non-federal features, Grievances, Government of India. The CBI is
viz., a strong Centre, single Constitution, single governed by The Delhi Special Police
citizenship, flexibility of Constitution, integrated Establishment (DSPE) Act, 1946.
judiciary, appointment of state governor by the * It must mandatorily obtain the consent of the state
Centre, all-India services, emergency provisions government concerned before beginning to
and so on. investigate a crime in a state. The state
government’s consent to CBI can be either case-
specific or general.
40) Consider the following statements regarding
the Central Bureau of Investigation (CBI): * General consent is usually given by states to help
1) It was set up on recommendation of the the CBI in the seamless investigation of cases of
Tarapore Committee. corruption against central government employees
in their states. This is consent by default, in the
2) It functions under the Ministry of Personnel, absence of which the CBI would have to apply to
Pension & Public Grievances. the state government in every case, and before
3) It must mandatorily obtain the consent of taking even small actions.
the state government concerned before
beginning to investigate a crime in a state.
41) Consider the following statements with
Which of the statements given above is/are reference to the Montagu-Chelmsford
correct? Reforms of 1919 :
a) 1 and 2 only 1. It provided universal adult suffrage.
b) 2 and 3 only 2. It provided for the establishment of a public
c) 3 only service commission.
d) 1, 2 and 3 Which of the statements given above is/are
correct?
Solution (b)
a) 1 only
* Recently, Bihar’s newly formed government called
for withdrawing the general consent to the CBI. b) 2 only
* Central Bureau of Investigation (CBI) is the c) Both 1 and 2
premier investigating police agency in India. It has d) Neither 1 nor 2
KPIAS (21 - D) Cell: 91332 37733
Answer : b
Explanation : 43) Consider the following statements regarding
the Electoral Bonds:
* There was no universal adult suffrage under the
Montagu-Chelmsford reforms (the Government 1) An Electoral Bond is like a promissory note.
of India Act, 1919 ). 2) It can be purchased in cash or digitally
* Government of India Act, 1919 provided for the through any commercial bank.
establishment of a public service commission. 3) All the Political Parties are eligible to receive
the Electoral Bonds.

42) Which among the following statements show Which of the given above statements is/are
that the Indian Constitution is a synthesis of correct?
Parliamentary Sovereignty and Judicial a) 1 only
Supremacy.
b) 3 only
1. Article 368 of the Constitution.
c) 1 and 2 only
2. Article 13 of the Constitution.
d) 1,2 and 3
Select the correct answer using the code given
Solution (a)
below.
* An Electoral Bond is like a promissory note that
a) 1 only
may be purchased by a person who is a citizen of
b) 2 only India or incorporated or established in India.
c) Both 1 and 2 * One can purchase these bonds only digitally or
d) Neither 1 nor 2 through cheques. As of now, only SBI is
authorised to issue the Electoral Bonds.
Answer : c
* Only the Political Parties registered under Section
Explanation: 29A of the Representation of the People Act
Article 368 provides the power to amend the (RPA), 1951 and which secured not less than one
Constitution. Parliament can amend any part of percent of the votes polled in the last General
the constitution. But this power is not unlimited Election to the House of the People or the
as, according to Article 13(2), parliament cannot Legislative Assembly of the State, shall be eligible
enact any law which takes away or abridges the to receive the Electoral Bonds.
fundamental rights. This power limits the
sovereignty of the Parliament.
44) Consider the following statements with
* Hence, Article 368 and Article 13 show that the reference to the ‘Government of India Act
Indian Constitution is a synthesis of Parliamentary 1935’ :
Sovereignty and Judicial Supremacy.
1. The Act divided the powers between the
Centre and the Provinces into three lists.
KPIAS (22 - D) Cell: 91332 37733
2. It abolished dyarchy in the provinces and
Centre. However, this provision of the Act did not
introduced ‘provincial autonomy’ in its
come into operation at all.
place.
4. It introduced bicameralism in six out of eleven
3. It provided for bicameral legislature in all
provinces.
the provinces of British India.
5. It further extended the principle of communal
4. The Government of India Act of 1935 gave
representation by providing separate electorates
women reserved seats in the legislature.
for depressed classes (scheduled castes), women
Select the correct answer using the code given and labour (workers).
below.
6. It provided for the establishment of a Reserve
a) 1, 2 and 4 only Bank of India to control the currency and credit
b) 1 and 3 only of the country.
c) 3 only 7. It provided for the establishment of not only a
Federal Public Service Commission but also a
d) 1, 2, 3 and 4
Provincial Public Service Commission and Joint
Answer : a Public Service Commission for two or provinces.
Explanation : 8. It provided for the establishment of a Federal Court,
Features of the Government of India 1935 Act : which was set up in 1937.

1. It provided for the establishment of an All India 9. The Government of India Act of 1935 gave
Federation consisting of provinces and princely women reserved seats in the legislature.
states as units. The Act divided the powers
between the Centre and units in terms of three
45) Consider the following statements on
lists— Federal List (for Centre, with 59 items),
functions of special officer on linguistic
Provincial List (for provinces, with 54 items) and
minority
the Concurrent List (for both, with 36 items).
Residuary powers were given to the Viceroy. 1) To investigate all matters related to
However, the federation never came into being safeguards provided to the linguistic
as the princely states did not join it. minorities.
2. It abolished dyarchy in the provinces and 2) Report to President of India regarding status
introduced ‘provincial autonomy’ in its place. of implementation of the Constitutional and
Moreover, the Act introduced responsible the nationally agreed Scheme of Safeguards
governments in provinces, that is, the governor was
required to act with the advice of ministers
responsible to the provincial legislature. This came
into effect in 1937 and was discontinued in 1939.
3. It provided for the adoption of dyarchy at the

KPIAS (23 - D) Cell: 91332 37733


for the linguistic minorities. for fixation/revision of tariffs for telecom
services.
3) Monitoring status of implementation of
Safeguards through Questionnaires, Visits, 2) It provides a fair and transparent policy
Conferences. environment which promotes a level playing
field.
Which of the following statements is/are
correct? Which of the statements given above is/are
correct?
a) 1 only
a) 1 only
b) 1 and 2 only
b) 2 only
c) 2 and 3 only
c) Both 1 and 2
d) All of the above
d) Neither 1 nor 2
Solution (d)
Solution (b)
Functions of commissioner for linguistic minorities
in India: * Recently, the Telecom Regulatory Authority of
India (TRAI) released its recommendations on
* To investigate all matters related to safeguards ‘Regulatory Framework for Promoting Data
provided to the linguistic minorities. Economy Through Establishment of Data Centres
* Report to President of India regarding status of (DCs), Content Delivery Networks (CDNs), and
implementation of the Constitutional and the Interconnect Exchanges (IXPs) in India’.
nationally agreed Scheme of Safeguards for the * The Telecom Regulatory Authority of India
linguistic minorities. (TRAI) was established with effect from 20th
* Monitoring status of implementation of Safeguards February 1997 by an Act of Parliament, called the
through Questionnaires, Visits, Conferences, Telecom RegulatoryAuthority of India Act, 1997,
Seminars, Meetings, Review Mechanism, etc. to regulate telecom services, including fixation/
revision of tariffs for telecom services which were
* Equality before law (Article 14), prohibition of
earlier vested in the Central Government.
discrimination on grounds of religion, race, caste,
sex or place of birth (Article 15) and equality of * TRAI’s mission is to create and nurture conditions
opportunity in matters of public employment for growth of telecommunications in the country
(Article 16) also operate as safeguards for linguistic in a manner and at a pace which will enable India
minorities. to play a leading role in emerging global
information society.
* One of the main objectives of the TRAI is to
46) With reference to Telecom Regulatory
provide a fair and transparent policy environment
Authority of India (TRAI), consider the
which promotes a level playing field and facilitates
following statements:
fair competition.
1) It was established by an Executive order
KPIAS (24 - D) Cell: 91332 37733
48) With reference to Vice President of India,
47) Consider the following statements relating consider the following statements:
to fundamental rights:
1) The Vice-President of India is elected by
1. Under the President’s rule the President can an electoral college consisting of members
suspend the enforcement of any or all the of Rajya Sabha only.
fundamental rights, except the fundamental
rights given under Articles 20 and 21. 2) The Election Commission of India conducts
the election to the office of the Vice-
2. Article 29 promotes harmonious diversity President.
in the country.
3) All doubts and disputes arising in connection
Which of the statements given above is/are with the election of the Vice-President are
correct? enquired by the Supreme Court of India.
a) 1 only Which of the statements given above is/are
b) 2 only correct?
c) Both 1 and 2 a) 1 and 2
d) Neither 1 nor 2 b) 2 only
Answer: b c) 1 and 3
Explanation: d) 2 and 3 only
* When a National Emergency is proclaimed but Solution (d)
not under President’s Rule, the President under * Recently, the Election Commission has held the
Article 359, can suspend the enforcement of any election to the office of the vice-president of India.
or all the fundamental rights, except the Right to
Protection in respect of conviction for offences * The Vice-President of India is the second highest
(Article 20) and Right to life and personal liberty constitutional office in the country. He serves for
(Article 21). a five-year term, but can continue to be in office,
irrespective of the expiry of the term, until the
* Article 29 recognises the fundamental right of any successor assumes office.
section of the citizens, resident in India, the right
to conserve its distinct identity based on language, * The Vice-President is elected by an electoral
script or culture. The term “Culture” includes college consisting of members of both Houses of
identity based on religion as well. Therefore, Parliament, in accordance with the system of
Article 29 seeks to promote pluralism, tolerance proportional representation by means of the single
and co- existence among the people. The transferable vote and the voting in such election is
acceptance of the above concepts is the core by secret ballot (Article 66 (1)).
of India’s unity that is built on its diversity. * The Election Commission of India conducts the
Thus Article 29 promotes harmonious election to the office of the Vice-President. The
diversity. Election Commission is mandated to ensure that

KPIAS (25 - D) Cell: 91332 37733


50) Which among the following is called as
the election to the office of the Vice-President of
“Conscience of the constitution” by Granville
India must be a free and fair election and the
Austin?
Commission needs to take all necessary steps for
discharging its constitutional responsibility. 1. Fundamental Rights.

* All doubts and disputes arising in connection with 2. Fundamental Duties.


the election of the Vice-President are enquired 3. Directive Principles of State Policy
into and decided by the Supreme Court of India
Choose the correct codes
whose decision is final. A petition challenging the
election of the Vice-President is heard by a five- a) 1 and 2 only
judge bench of the Supreme Court of India. b) 1 only
* Jagdeep Dhankhar has been elected as the 14th c) 1 and 3 only
Vice-President of India.
d) All the above
Answer: c
49) Parliamentary System in India was introduced
for the first time under which of the following: Explanation:

a) Government of India Act, 1935 * The Directive Principles along with the
Fundamental Rights contain the philosophy of the
b) Government of India Act, 1858 Constitution and is the soul of the Constitution.
c) Government of India Act, 1919 Granville Austin has described the Directive
Principles and the Fundamental Rights as the
d) Charter Act of 1853
‘Conscience of the Constitution’
Answer : d
Explanation :
51) Which of the following can cast a vote during
* The Charter Act of 1853 separated, for the first elections in India?
time, the executive and legislative functions of the
1) Undertrial Prisoners
Governor General’s council. It provided for the
addition of 6 legislative councilors to the council. 2) Convicts out on bail
* Hence, it created a separate Governor General’s 3) People under preventive detention
legislative council which came to be known as the 4) Convicts serving their sentence in jails
(Indian Central Legislative Council). It functioned
as a mini parliament. Select the correct answer using the code
given below:
* Hence, the Charter Act of 1853 introduced
Parliamentary System in India for the first time a) 1 only
b) 2 and 3 only
c) 2 and 4 only

KPIAS (26 - D) Cell: 91332 37733


d) None of the above
Solution (b)
* void ab initio but become unenforceable only
Following can cast vote in India: to the extent of such inconsistency with
* While convicts out on bail could vote, under trials, the fundamental rights.
whose innocence or guilt has not been conclusively
* This is dealt under Article 13(1) of the Indian
determined, and those confined in civil persons
Constitution.
were deprived of their right to vote.
* Doctrine of Severability can be explained as-
* Recently a petition challenged the provision in the
an act may not be void as a
election law that imposes a blanket ban on under
trials, persons confined in civil prisons and convicts * whole but only a part of it may be void and if that
serving their sentence in jails from casting their part is severable from the
votes. * rest which is valid, and then the rest may continue
* While prisoners are not allowed to vote, people to stand and remain operative. Under article
under preventive detention can cast their votes 13, it is given that only offending part in a statute
through postal ballots. is void and not the whole statute.
Note: The doctrine of eclipse does not apply to post-
constitutional laws (Article 13(2)) as they are void
52) A law which violates fundamental rights, is
from their very inception.
not nullity or void ab-initio but becomes, only
unenforceable.
The doctrine discussed above is 53) With the commencement of the Constitution
of India, under Article 395, which of the
a) Doctrine of Repugnancy
following Acts were repealed?
b) Doctrine of Waiver
1. India Independence Act of 1947
c) Doctrine of Eclipse
2. Government of India Act of 1935
d) Doctrine of Severability
3. Government of India Act of 1919
Answer: c
4. Indian councils act of 1909.
Explanation:
Select the correct answer using the code given
* Article 13 of the Indian Constitution describes the below.
means for judicial review.
a) 1 and 2 only
* Doctrine of Eclipse provides for the validation
b) 1 and 3 only
of Pre-Constitution Laws that
c) 1 only
* violate fundamental rights upon the premise
that such laws are not null and

KPIAS (27 - D) Cell: 91332 37733


• EVMs were first used in Parur Assembly
d) 2, 3 and 4 only Constituency of Kerala in the year 1982.
Answer : a • In 1988, a new section 61A was inserted in
Explanation : the Representation of the People Act, 1951
empowering ECI to use electronic voting
With the commencement of the constitution of
India, The India Independence Act of 1947 and machines.
Govt. of India Act of 1935 were repealed because • An EVM being used by ECI can record a
the India Independence Act of 1947 itself provided maximum of 2,000 votes. In the old versions
for repeal of any act of British Parliament including (2000-05) model, a maximum of 3,800 votes
India Independence Act of 1947 itself. could be cast
• EVMs can cater to a maximum of 64
candidates (including NOTA). There is
54) With reference to Electronic voting machine
provision for 16 candidates in one Balloting
(EVM), consider the following statements.
Unit.
1. EVMs were first used in Parur Assembly
• The arrangement of names of candidates
Constituency of Kerala in the year 1982.
in the ballot paper is in alphabetical order,
2. An EVM being used by ECI can record a first candidates from national political
maximum of 2,000 votes. parties’ figure and then from other state
3. EVMs can cater to a maximum of 64 registered parties.
candidates.
Select the correct answer using the code 55) Consider the following statements regarding
given below. minorities in India:
a) 1 and 3 only 1) A Special Officer for linguistic minorities is
b) 2 and 3 only appointed by the National Commission for
Minorities.
c) 1 and 2 only
2) All members of the National Commission
d) 1, 2 and 3
for Minorities must belong to communities
Answer: d enlisted as minorities.
Explanation: EVM is a microcontroller-based portable Which of the statements given above is/are
instrument designed to modernise the election correct?
procedure of conducting elections.
a) 1 only
• EVM consists of two Units – a Control Unit
b) 2 only
and a Balloting Unit. These units are joined
together by a cable. It works on normal c) Both 1 and 2
batteries and does not consume electricity. d) Neither 1 nor 2

KPIAS (28 - D) Cell: 91332 37733


Solution (b) 3. It provided that the Viceroy could nominate
Indians to his legislative council.
* Article 350 B says there shall be a Special Officer
for linguistic minorities to be appointed by the 4. It provided for the establishment of a Central
President. Public Service Commission.
National Commission for Minorities (NCM)
* In 1978, the setting up of the Minorities Select the correct answer using the code given
Commission (MC) was envisaged in the Ministry below :
of Home Affairs Resolution. In 1984, the MC was a) Only one of the statements is correct.
detached from the Ministry of Home Affairs and
placed under the newly created Ministry of b) Only two of the statements are correct.
Welfare, which excluded linguistic minorities from c) Only three of the statements are correct.
the Commission’s jurisdiction in 1988. In 1992, with
d) All four statements are correct.
the enactment of the NCM Act, 1992, the MC
became a statutory body and was renamed as the Answer : c
NCM. Explanation :
* The first National Commission for Minorities with Features of the Indian Councils Act of 1861 :
a statutory status was formed in 1993.
* It made a beginning of representative institutions
Composition of the commission: by associating Indians with the law-making
* The commission consists of seven members which process. It thus provided that the viceroy should
include a Chairperson and a Vice-Chairperson nominate some Indians as non-official members
along with five other members. of his expanded council. In 1862, Lord Canning,
the then viceroy, nominated three Indians to his
* All members of the commission must belong to
legislative council-the Raja of Benaras, the
minority communities enlisted as minorities, and
Maharaja of Patiala and Sir Dinkar Rao.
from amongst persons of eminence, ability and
integrity. Every member of the Commission holds * It initiated the process of decentralisation by
office for a period of three years. restoring the legislative powers to the Bombay and
Madras Presidencies. It thus reversed the
centralising tendency that started from the
56) Consider the following statements with Regulating Act of 1773 and reached its climax
reference to the Indian Councils Act of 1861 under the Charter Act of 1833. This policy of
: legislative devolution resulted in the grant of almost
1. It provided for the establishment of new complete internal autonomy to the provinces in
legislative councils for Bengal and Punjab. 1937.

2. It empowered the Viceroy to issue * It also provided for the establishment of new
ordinance without concurrence of the legislative councils for Bengal, North-Western
legislative council. Frontier Province (NWFP) and Punjab, which
KPIAS (29 - D) Cell: 91332 37733
c) 1 and 3 only
were established in 1862, 1866 and 1897 d) 3 only
respectively.
Solution (b)
* It empowered the Viceroy to make rules and
orders for the more convenient trans-action of * To avoid conflict of duty, there are a few limitations
business in the council. It also gave a recognition that are posted on the Attorney General which he
to the ‘portfolio’ system, introduced by Lord should keep in mind while performing his duties:
Canning in 1859. Under this, a member of the * He should not advise or hold a brief against the
Viceroy’s council was made in-charge of one or Government of India.
more departments of the government and was
* He should not advise or hold a brief in cases in
authorised to issue final orders on behalf of the
which he is called upon to advise or appear for
council on matters of his department(s).
the Government of India.
* It empowered the Viceroy to issue ordinances,
* He should not defend accused persons in criminal
without the concurrence of the legislative council,
prosecutions without the permission of the
during an emergency. The life of such an
Government of India.
ordinance was six months.
* He should not accept appointment as a director in
* Government of India Act, 1919 provided for the
anycompanyor corporation without the permission
establishment of a public service commission.
of the Government of India.
Hence, a Central Public Service Commission was
set up in 1926 for recruiting civil servants.
Hence, statements 1, 2 and 3 are correct. 58) Consider the following statements regarding
‘Quo-Warranto’:
1. It prevents illegal usurpation of public office
57) With reference to Attorney General of India
by a person.
consider the following statements
2. It cannot be issued in case of Ministerial
1) He can advise or hold a brief against the
office or private office.
Government of India.
3. This can be sought by any interested person
2) He should not defend accused persons in
and not necessarily by the aggrieved person.
criminal prosecutions without the permission
of the Government of India. Which of the above statements are correct?
3) He should not accept appointment as a a) 1 and 2
director in any company or corporation b) 1 and 3
without the permission of the President.
c) 2 and 3
Which of the following statements is correct?
d) All of the above
a) 1 and 2 only
Answer: d
b) 2 only
KPIAS (30 - D) Cell: 91332 37733
Explanation: d) Neither 1 nor 2
* In the literal sense, quo-warranto means ‘by what Answer: d
authority or warrant’. It is issued by the court to Explanation: Both statements are INCORRECT.
enquire into the legality of the claim of a person to
a public office. Hence, it prevents illegal usurpation * The 103rd Amendment inserted Articles 15(6)
of public office by a person. and 16(6) in the Constitution to provide up to 10
per cent reservation to the economically weaker
* In other words, if the court finds that a person is sections (EWS) among non-OBC and non-SC/
holding office but is not entitled to hold that office, ST sections of the population.
it issues the writ of quo warranto and restricts
that person from acting as an office holder. * The amendment had changed the Constitution and
introduced a quota for the poor among the so-
* The writ of quo warranto applies in the case of an
called ‘forward castes’ or ‘general category’.
office which is public and not private in nature,
i.e., established by law or the Constitution. The * The quota is available in admissions to higher
public office must be substantive in nature, which educational institutions, and in initial recruitment
excludes mere employment or function of a servant in central government jobs. The amendment also
at the pleasure of another. empowered state governments to provide
reservation on the basis of economic
* There is no bar or restriction on who can apply. backwardness.7
Any person can apply as long as their fundamental
or any other legal right is being breached. In cases
where there is no breach of right, a question of 60) Consider the following statements regarding
public interest must arise with respect to the the Government of India Act of 1919 :
application.
1. The Act provided for a Preamble.
2. According to the act, the policy of the
59) Consider the following statements: British Parliament was to develop self
1. 10% quota for economically weaker governing institutions with a view to the
sections (EWS) was introduced through the progressive realisation of responsible
102nd ConstitutionalAmendment. government in British India as an integral
part of the empire.
2. Members of SC/ST/OBC can also avail the
EWS reservation. Which of the statements given above is/are
correct?
Select the correct answer using the code given
below. a) 1 only

a) 1 only b) 2 only

b) 2 only c) Both 1 and 2

c) Both 1 and 2 d) Neither 1 nor 2


Answer : c
KPIAS (31 - D) Cell: 91332 37733
Explanation : * Political and administrative consideration-Delhi and
* The Government of India Act of 1919 was enacted Chandigarh.
to satisfy the people of India to some extent. The * Cultural distinctiveness-Puducherry, Dadra and
Act provided for a Preamble that laid down the Nagar Haveli, and Daman and Diu.
basic principles and policies upon which it was
* Strategic importance-Andaman and Nicobar
based.
Islands and Lakshadweep.
According to it the policy of the British Parliament * Special treatment and care of the backward and
was : tribal people-Mizoram, Manipur, Tripura and
i) to provide for the increasing association of Indians Arunachal Pradesh which later became states.
in every branch of Indian administration,
ii) to develop self governing institutions with a view
62) Which among the following directive
to the progressive realisation of responsible
Principles were not added by 42nd amendment
government in British India as an integral part of
of Constitution?
the empire;
a) Secure the opportunities for healthy
iii) accordingly, the Preamble suggested for a development of Children.
decentralised unitary form of government.
b) Promote equal justice and free legal aid to
the poor.
61) Consider the following statements : c) Minimize inequalities in income, status,
1. Cultural distinctiveness facilities and opportunities.
2. Strategic importance d) To take steps to secure the participation of
3. Interest of tribal people workers in the management of industries.

4. Administrative consideration Answer: c


Explanation:
Which of the above is / are the reasons for creation
of Union Territories in India ? * The 42nd Amendment Act of 1976 added four new
Directive Principles to the original list:
a) 1, 3 and 4 only
* To secure opportunities for healthy development
b) 2 and 4 only
of children (Article 39(f))
c) 2, 3 and 4 only
* To promote equal justice and to provide free legal
d) 1, 2, 3 and 4 aid to the poor (Article 39A)
Answer : d * To take steps to secure the participation of workers
Explanation : in management of industries (Article 43A)

The reasons for creation of the Union Territories * To protect and improve the environment and to
in India are : safeguard forests and wildlife (Article 48A)

KPIAS (32 - D) Cell: 91332 37733


* 44th Constitutional Amendment, 1978: It inserted * The Rajya Sabha, constitutionally the Council of
Section-2 to Article 38 which declares that; “The States, is the upper house of the bicameral
State in particular shall strive to minimise economic Parliament of India.. There is representation of
inequalities in income and eliminate inequalities in states in the Rajya sabha. So the Rajya sabha
status, facilities and opportunities not amongst protects and promotes the interest of the states at
individuals but also amongst groups”. It also the national level. It also checks the excessive
eliminated the Right to Property from the list of power enjoyed by the central government.
Fundamental Rights. Hence Option C is
INCORRECT.
64) Which one of the following Directive
principles was not originally provided in the
63) Cooperative Federalism can enable good constitution?
governance in India. To actualise this a) Uniform civil code
important goal, which of the following
provisions are incorporated in the b) Protection of monuments
Constitution of India ? c) Provision of Living Wage
1. Inter-State Council d) Free legal aid
2. Integrated judicial system. Answer: d
3. The Council of states. Explanation:
Select the correct answer using the code given * The Directive Principle directing the state – to
below. promote equal justice and to provide free legal aid
a) 1 and 2 only to the poor (Article 39A) was added by the 42nd
Amendment Act of 1976.
b) 2 only
Article 44: Uniform civil code
c) 2 and 3 only
Article 49: Protection of monuments
d) 1, 2 and 3
Article 39- Provision for living wage
Answer : d
Explanation :
65) With reference to the Comptroller and
* The Inter-State Council was created on the Auditor General of India, consider the
recommendation of Sarkaria commission. It helps
following statements:
in resolving disputes between the Centre and
States and so promote cooperation between them. 1) He is entrusted with auditing the accounts
of District and Regional Councils of tribal
* In India there is an integrated judicial system. The areas.
single system of courts enforces both the central
and state laws. So there is no division of judicial 2) The propriety audit is discretionary on the
powers between supreme court and high courts part of the CAG.

KPIAS (33 - D) Cell: 91332 37733


Which of the statements given above is/are * In addition to the legal and regulatory audit, the
correct? CAG can also conduct the propriety audit, i.e., he
can look into the wisdom, faithfulness and economy
a) 1 only
of government expenditure and comment on the
b) 2 only wastefulness and extravagance of such
c) Both 1 and 2 expenditure. However, unlike the legal and
regulatory audit, which is obligatory on the part of
d) Neither 1 nor 2
the CAG, the propriety audit is discretionary.
Solution (c)
* As a constitutional functionary, CAG is primarily
66) Consider the following statements with
entrusted with the responsibility to audit the
reference to the Charter Act of 1833?
accounts and related activities of the three tiers
of Government – Federal, Provincial and Local; 1. The act ended the commercial activity of
the State-owned public sector commercial East India Company and reduced it to an
enterprises; and autonomous bodies financed by administrative body.
the Federal and Provincial Governments. His 2. It made the Governor-General of Bengal
reports are laid before the Parliament and as the Governor-General of India and
Legislatures of the Provinces. vested in him all civil and military powers.
District and Regional Funds: 3. The Act for the first time, created a
* There shall be constituted for each autonomous Government of India having authority over
district, a District Fund and for each autonomous the entire territorial area possessed by the
region, a Regional Fund. British in India.
* The Governor may make rules for the Select the correct answer using the code given
management of the District Fund, or, as the case below.
may be, the Regional Fund. a) 1 and 2 only
* The accounts of the District Council or, as the b) 1 only
case may be, the Regional Council shall be kept in
such a form as the Comptroller and Auditor c) 2 only
General of India may, with the approval of the d) 1,2 and 3.
President prescribed. Answer : d
* The Comptroller and Auditor-General shall cause Explanation :
the accounts of the District and Regional Councils
to be audited in such manner as he may think fit, * The act ended the activities of the East India
and the reports of the Comptroller and Auditor- Company as a commercial body, which became a
General relating to such accounts shall be purely administrative body. It provided that the
submitted to the Governor who shall cause them company’s territories in India were held by it ‘in
to be laid before the Council. trust for His Majesty, His heirs and successors.

KPIAS (34 - D) Cell: 91332 37733


* This act made the Governor-General of Bengal 2. It provided for the establishment of a
as the Governor-General of India and vested in Supreme Court at Calcutta.
him all civil and military powers. Thus, the act
Which of the statements given above is/are
created, for the first time, a Government of India
correct?
having authority over the entire territorial area
possessed by the British in India. Lord William a) 1 only
Bentick was the first governor-general of India. b) 2 only
c) Both 1 and 2
67) Consider the following statements regarding d) Neither 1 nor 2
the Indian Councils Act of 1861:
Answer : c
1. The act empowered the Viceroy to issue
Explanation :
ordinances without the approval of the
Legislative Council. The Regulating Act of 1773 :
2. It introduced the office of Secretary of * It designated the Governor of Bengal as the
State for India. ‘Governor-General of Bengal’ and created an
Executive Council of four members to assist him.
Which of the statements given above is/are
The first such Governor-General was Lord Warren
correct?
Hastings. It made the governors of Bombay and
a) 1 only Madras presidencies subordinate to the Governor
b) 2 only General of Bengal, unlike earlier, when the three
presidencies were independent of one another.
c) Both 1 and 2
d) Neither 1 nor 2
* It provided for the establishment of a Supreme
Answer : a Court at Calcutta (1774) comprising one chief
Explanation : justice and three other judges.
Indian Councils Act of 1861 empowered Viceroys * The act also prohibited the servants of the
to issue ordinances without concurrence of the Company from engaging in any private trade or
legislative council, during an emergency. The new accepting presents or bribes from the ‘natives’.
office of Secretary of State for India was created
by the Government of India Act of 1858.
69) Consider the following statements regarding
the Presidential system of government :
68) Consider the following statements with 1. In this system, the President is accountable
reference to the Regulating Act of 1773 : to the legislature.
1. The governors of Bombay and Madras 2. In this system, the President is the head of
presidencies were made subordinate to the government.
Governor-general of Bengal.
KPIAS (35 - D) Cell: 91332 37733
Which of the statements given above is/are * Dyarchy means dual set of governments, e.g
correct? accountable and nonaccountable. In the
transferred subjects the Governors were to be
a) 1 only
assisted by the ministers responsible to the
b) 2 only legislature while in the reserved subjects the
c) Both 1 and 2 Governors were to be advised by the councillors
who were not accountable to the legislature.
d) Neither 1 nor 2
Answer : b
71) Consider the following Statements regarding
Explanation :
the Charter Act of 1853 :
* In the Presidential System, the President is not
1. For the first time there was separation of
accountable to the legislature.
the legislative and executive functions of
* Also, the President is head of the state as well as the Governor General’s council
the head of the government.
2. The Act introduced an open competition
system for selection and recruitment of civil
70) The principle of ‘Dyarchy’ enacted under the servants.
Government of India Act of 1919 refers to : 3. The act deprived the Governor of Bombay
a) Division of the central legislature into two and Madras of their legislative powers.
houses. 4. In accordance with the act the Committee
b) Division of powers between the Central and on the Indian Civil Service was appointed.
Provincial governments. Which of the statements given above is/are
c) Separation of powers between British India correct?
and the Princely states. a) 1, 2 and 4 only
d) Division of the subjects delegated to the b) 2 and 3 only
provinces into two categories.
c) 1 only
Answer : d
d) 2, 3 and 4 only
Explanation :
Answer : a
* The Government of India Act of 1919 divided the
Explanation :
functions of government in two categories: central
and provincial. The Charter Act of 1853 :
* The provincial subjects were further subdivided * It separated, for the first time, the legislative and
into transferred and reserved. Thus, in the executive functions of the Governor General’s
provinces a new form of government, dyarchy, council. It provided for the addition of six new
was introduced. members called legislative councilors to the
council. In other words, it established a separate
KPIAS (36 - D) Cell: 91332 37733
Governor-General’s legislative council which came d) Neither 1 nor 2
to be known as the Indian (Central) Legislative Answer : c
Council.
Explanation :
* This legislative wing of the council functioned as
a mini- Parliament, adopting the same procedures The Indian Independence Act of 1947 made the following
as the British Parliament. Thus, legislation, for the changes in the position of the Constituent Assembly:
first time, was treated as a special function of the 1. The Assembly was made a fully sovereign body,
government, requiring special machinery and which could frame any Constitution it pleased. The
special process. act empowered the Assembly to abrogate or alter
* The Act of 1853 introduced an open competition any law made by the British Parliament in relation
system of selection and recruitment of civil to India.
servants. The covenanted civil services was thus 2. The Assembly also became a legislative body. In
thrown open to the Indians also. other words, two separate functions were assigned
* Accordingly, the Macaulay Committee (the to the Assembly, that is, making of a constitution
Committee on the Indian Civil Service) was for free India and enacting of ordinary laws for
appointed in 1854. the country. These two tasks were to be performed
on separate days.
* The Charter Act of 1833 deprived the Governor
of Bombay and Madras of their legislative powers. 3. Thus, the Assembly became the first Parliament
The Governor General of India was given exclusive of free India (Dominion Legislature). Whenever
legislative powers for the entire British India. the Assembly met as the Constituent body it was
chaired by Dr. Rajendra Prasad and when it met
as the legislative body, it was chaired by G V
72) Consider the following statements with Mavlankar. These two functions continued till
reference to the Indian Independence Act, November 26, 1949, when the task of making the
1947 : Constitution was over.
1. The constituent assembly was made a fully
sovereign body to frame any Constitution.
73) Consider the following statements regarding
2. The constituent assembly was also granted the Indian Independence Act, 1947 ?
the legislative function of enacting ordinary 1. It ended the British rule in India and declared
laws for India. India as an independent and sovereign state
Select the correct answer using the code given from August 15, 1947.
below. 2. The act abolished the office of Governor
a) 1 only General in India.
b) 2 only 3. The act did not proclaim the lapse of British
c) Both 1 and 2 Paramountcy over Indian Princely states.
4. The act provided that India can alter any
KPIAS (37 - D) Cell: 91332 37733
law made by the British in relation to India. 20(3) – ‘No person accused of an offence shall
Select the correct answer using the code given be compelled to be a witness against himself’.
below. * The protection against self-incrimination extends
a) 1 only to both oral evidence and documentary evidence.
But it does not extend to compulsory production
b) 1 and 4 only of material objects, compulsion to give thumb
c) 3 and 4 only impression, blood specimen and compulsory
exhibition of the body. Hence Statement 1 is
d) 1, 2 and 3 only
INCORRECT.
Answer : b
* It extends only to criminal proceedings and not to
Explanation : civil proceedings or
By the India Independence act 1947, the * proceedings which are not criminal in nature.
constituent assembly was made a fully sovereign Hence Statement 2 is INCORRECT.
body. This act also empowered the assembly to
abrogate or alter any law made by the British in
relation to India . The act abolished the office of 75) Consider the following statements:
viceroy and provided for each dominion, a 1) Attorney General for India is appointed by
governor - General. This act ended British Rule in the President from among persons who are
India and declared India as an independent and “qualified to be appointed as a Judge of the
sovereign state from August 15, 1947. Supreme Court”.
2) The Attorney-General of India shall have
74) Consider the following statements regarding the right to take part in the proceedings of
protection against self-incrimination Rajya Sabha only.
1. It extends only to oral evidence and not to Which of the statements given above are
documentary evidence. correct?
2. It extends to both civil and criminal a) 1 Only
proceedings. b) 2 Only
Select the correct answer using the code given c) Both 1 and 2
below:
d) Neither 1 nor 2
a) 1 only b) 2 only
Solution (a)
c) Both 1 and 2 d) Neither 1 nor 2
* Senior Advocate Mukul Rohatgi has declined the
Answer: d government’s offer to be Attorney General (A-G)
Explanation: for India.
* The Indian Constitution provides immunity to an
accused against self-incrimination under Article
KPIAS (38 - D) Cell: 91332 37733
* The A-G is the Government of India’s first law the states.
officer, and has the right of audience in all courts * Supreme Court and high courts.
of the country. Under Article 76(1), the A-G is
* Distribution of legislative powers between the
appointed by the President from among persons
who are “qualified to be appointed as a Judge of Union and the states.
the Supreme Court”. * Any of the lists in the Seventh Schedule.
* Under Article 88, the “Attorney-General of India * Representation of states in Parliament.
shall have the right to speak in, and otherwise to * Power of Parliament to amend the Constitution
take part in the proceedings of, either House, any and its procedure (Article368 itself)
joint sitting of the Houses, and any committee of
Parliament of which he may be named a member”. 77. With reference to National Educational
Alliance for Technology (NEAT), consider the
following statements:
76. Which of the following must be amended by 1. It is 100% government of India initiative.
Special Majority of Parliament and Consent
of the States? 2. The initiative provides for the use of best-
developed technological solutions in the
1. Election of the President and its manner education sector to enhance the
2. Distribution of legislative powers between employability of the youth.
the Union and the states 3. All India Council for Technical Education
3. Power of Parliament to amend the (AICTE) is acting as the facilitator in the
Constitution process.
Select the correct answer from the codes given Which of the statements given above is/are
below: correct?
a) 1 and 2 only a) 1 and 2 only
b) 2 and 3 only b) 2 only
c) 1 and 3 only c) 2 and 3 only
d) 1, 2 and 3 d) 1,2 and 3 only
Solution: D Answer: C
Explanation Explanation
By Special Majority of Parliament and Consent of States, * Statement 1 is incorrect: NEAT is a Public-Private
the following provisions can be amended in this way: Partnership model between the Government
(through its implementing agencyAICTE) and the
* Election of the President and its manner.
Education Technology companies of India.
* Extent of the executive power of the Union and

KPIAS (39 - D) Cell: 91332 37733


* Statement 2 is correct: National Educational a) 1 and 2 only
Alliance for Technology (NEAT) is an initiative to b) 2 only
provide the use of bestdeveloped technological
solutions in the education sector to enhance the c) 2 and 3 only
employability of the youth on a single platform for d) 1, 2 and 3 only
learners' convenience.
Answer: D
* Statement 3 is correct: All India Council for
Explanation:
Technical Education (AICTE) is acting as the
facilitator in the process while ensuring that the * Rani Velu Nachiyar was a queen of Sivaganga
solutions are freely available to a large number of estate from c. 1780-1790.
socially and economically backward students. * She was the first Indian queen to wage war with
the East India Company in India.
78. World Press Freedom Index is released by? * She is widely known as Veeramangai ("brave
woman").
a) Reporters without borders
b) Pacific Research Institute
80. With reference to Solar Hydrogen
c) International Federation of Journalists
Programme, consider the following
d) Reporters Committee for the Freedom of statements:
Press
1. It was launched during the International
Answer: a Solar Alliance (ISA) assembly.
Explanation : 2. The programme aims to produce emission-
* World Press Freedom Index is released by free fuel at $2 per kg, which is much lower
Reporters Without Borders. than the current $5 per kg.
3. International Solar Alliance (ISA) aims to
achieve $1 trillion global in solar investments
79. With reference to Rani Velu Nachiyar, by 2030.
consider the following statements:
Which of the statements given above is/are
1. She is the 18th century queen from correct?
Sivagangai district in Tamil Nadu.
a) 1 and 2 only
2. She created a women's only army called
Udaiyaal. b) 2 only

3. She was the first Indian queen to wage war c) 2 and 3 only
with the East India Company in India. d) 1,2 and 3 only
Which of the statements given above is/are Answer: D
correct?
KPIAS (40 - D) Cell: 91332 37733
Explanation: 3. The Supreme Court by majority opinion has
* During the assembly, two new programmes upheld the exclusion of the backward
were launched: Management of Solar PV panels classes from the scheme of the EWS quota.
& battery usage waste & Solar Hydrogen Which of the statements given above is/are
programme. The new Hydrogen initiative is aimed correct?
at enabling the use of solar electricity to produce
a) 1 and 2 only
hydrogen at a more affordable rate than what is
available currently (USD 5 per KG), by bringing it b) 3 only
down to USD 2 per KG. c) 1 and 3 only
d) 2 and 3 only
81. Hyderabad Declaration was recently seen in Answer: B
news. It is related to which of the following?
Explanation:
a) Climate change
* The Supreme Court has recentlyupheld the validity
b) e-Governance of the 103rd Amendment of the Constitution which
c) E-mobility provides 10% reservation for Economically
Weaker Sections (EWS) in admissions and
d) Infrastructure
government jobs. (Janhit Abhiyan v Union of India
Answer: B case).
Explanation * The 10 per cent reservation in public sector jobs
* In the 24th National Conference on e-Governance and education was meant to benefit those who do
the 'Hyderabad Declaration' on e-Governance was not fall under any other quota category like
adopted. The declaration aims to bring citizens and Scheduled Caste (SC), Scheduled Tribe (ST) or
governments closer through digital platforms and Other Backward Classes (OBC). Hence,
transform citizen services through the use of Statement 3 is correct.
technology. * The EWS quota is secular, and all religious
minorities are eligible for the same provided they
meet the criteria as laid out by the DoPT.
82. With reference to reservations for
Economically Weaker Sections (EWS),
consider the following statements. 83. With reference to Africa50 platform, consider
1. The Constitution (104th Amendment) Act the following statements:
allows for reservation in jobs in the public 1. All African countries are members of the
sector and education for the economically platform.
weaker strata of society.
2. It is an infrastructure investment platform
2. Religious minorities are ineligible under the that contributes to Africa's economic
EWS quota.
KPIAS (41 - D) Cell: 91332 37733
growth. Explanation:
3. Power Grid Corporation of India Limited * The 93rd Amendment Act of 2005, added Article
(POWERGRID) has signed a Joint 15(5) as per which the state is empowered to make
Development Agreement with Africa50, to any special provision for the advancement of any
pioneer Africa's first transmission PPP socially and educationally backward classes of
project in Nigeria. citizens or for the scheduled castes or scheduled
tribes regarding their admission to educational
Which of the statements given above is/are
institutions including private educational
correct?
institutions, whether aided or unaided by the state,
a) 1 and 2 only except the minority educational institutions.
b) 2 only
c) 2 and 3 only 85. With reference to the Atma Nirbhar Bharat
d) 1,2 and 3 only Rozgar Yojana (ABRY), consider the following
statements:
Answer: B
1. It will help in formalization of informal
Explanation: employment and create new employment
* Statement 1 is incorrect: Currently the platform opportunities during and post COVID-19
has 31 shareholders comprised of 28 African Pandemic.
countries. 2. Scheme is being implemented through the
* Statement 3 is incorrect: POWERGRID and Employees Provident Fund Organization
Africa50 in Joint Development Agreement, to (EPFO).
pioneer Africa's first transmission PPP project in Which of the statements given above are correct?
Kenya.
a) 1 only
b) 2 only
84. Which of the following amendments
empowered states to make special provisions c) Both 1 and 2
for Socially and Educationally Backward d) Neither 1 nor 2
Sections (SEBCs) and SC/ST in admissions
Answer: C
to educational institutions?
Explanation:
a) 80th Amendment
* Statement 1 is correct- It was launched to boost
b) 93rd Amendment
employment generation and to minimize socio-
c) 97th Amendment economic impact of COVID-19 pandemic. It aims
d) 77thAmendment to reduces the financial burden of the employers
of various sectors/industries and encourages them
Answer: B to hire more workers.
KPIAS (42 - D) Cell: 91332 37733
* Statement 2 is correct-This scheme is being facilities, counselling, education, skill development,
implemented through the Employees Provident economic linkages etc with the support of State
Fund Organization (EPFO). Governments/UTs/Local Urban Bodies, Voluntary
Organizations.

86. With reference to the SMILE-Support for * Statement 2 is correct- The Ministry of Social
Marginalized Individuals for Livelihood and Justice and Empowerment has formulated an
Enterprise, consider the following umbrella scheme "SMILE - Support for
statements: Marginalized Individuals for Livelihood and
Enterprise", which includes two sub-schemes -
1. It is aimed primarily towards rehabilitation
'Central Sector Scheme for Comprehensive
of BPL (below poverty line) families.
Rehabilitation for Welfare of Transgender Persons'
2. Scheme includes education of the and 'Central Sector Scheme for Comprehensive
beneficiaries as one of its provisions. Rehabilitation of persons engaged in the act of
3. It includes two sub-schemes - 'Central Begging'.
Sector Scheme for Comprehensive
Rehabilitation for Welfare of Transgender
87. With reference to the Design Linked
Persons' and 'Central Sector Scheme for
Incentive (DLI) Scheme, which of the
Comprehensive Rehabilitation of persons
following statements are correct?
engaged in the act of Begging'.
1. The scheme offers financial incentives for
Which of the statements given above are correct?
both domestic as well as foreign companies
a) 1 and 2 only
2. The scheme has two components - Chip
b) 2 only Design infrastructure support, Product
c) 2 and 3 only Design Linked Incentive

d) 1, 2 and 3 only 3. C-DAC (Centre for Development of


Advanced Computing), a scientific society
Answer- C operating under MeitY, will serve as the
Explanation nodal agency for implementation of the DLI
scheme.
* Statement 1 is incorrect- This is an umbrella
scheme of Ministry of Social Justice and Select the correct answer using the given code
Empowerment which covers several below:
comprehensive measures including welfare a) 1 and 2 only
measures for both transgender persons and
persons who are engaged in the act of begging. b) 2 and 3 only

* Statement 2 is correct-It has provisions focussing c) None of the above


extensively on rehabilitation, provision of medical d) 3 only

KPIAS (43 - D) Cell: 91332 37733


Answer- D
c) 3 only
Explanation
d) 1 only
* Statement 1 and 2 are incorrect- Ministry of
Answer: D
Electronics and Information Technology has
invited applications from 100 domestic companies, * Article 371 of the Indian Constitution is connected
micro, small and medium enterprises (MSMEs) to granting special provisions for some states of
as well as startups for incentives under the design the Indian Union. As part of XXI and XXII of the
linked incentive (DLI) scheme. (Not Foreign Constitution of India, Article 371 grants some
companies). An applicant must meet the Threshold temporary, transition and special provisions to some
and Ceiling Limits to be eligible for disbursement states in the country. Ranging from Article 371-A
of incentives under the Scheme. to Article 371-J, this Article gives special
provisions for the states of Maharashtra, Gujarat,
* The scheme has three components - Chip Design
Nagaland, Assam, Manipur, Telangana, Andhra
infrastructure support, Product Design Linked
Pradesh, Sikkim, Mizoram, Arunachal Pradesh,
Incentive and Deployment Linked Incentive.
Goa and Karnataka.
* Statement 3 is correct-Centre for Development
* The Indian Constitution has certain unitary
of Advanced Computing (C-DAC) will act as the
tendencies. While other federations like USA
nodal agency for the implementation of the DLI
provide for dual citizenship, the India Constitution
scheme.
provides for single citizenship.
88. Consider the following statements:
* If a person has been sentenced to imprisonment
1. Article 371 A to 371 I were inserted in the for a period of 2 years or more, within five years
Constitution of India to meet regional of his registration or naturalization, his citizenship
demands of states like Nagaland, Assam, is terminated through "Deprivation".
Manipur,Andhra Pradesh, Sikkim, Mizoram,
Arunachal Pradesh and Goa.
89. With reference to National Emergency,
2. Constitutions of India and the United States
consider the following statements:
of America can envisage a dual policy (The
Union and the States) but a single 1. The proclamation of a war emergency
citizenship. cannot be made by the President unless the
Union cabinet gives him in written that such
3. A naturalized citizen of India can never be
proclamation should be made.
deprived of his citizenship.
2. The 38th Amendment Act of 1975 made
Which of the statements given above is/are
the declaration of a National Emergency
correct?
immune from the judicial review
a) 1, 2 and 3
3. The proclamation survives until 14 days
b) 1 and 3 from the first sitting of the Lok Sabha

KPIAS (44 - D) Cell: 91332 37733


Which of the statements given above is/are Article 19 during Emergencies
correct?
4. Article 360 - Provisions related to Financial
a) 1 only Emergency
b) 1 and 2 only Which of the above is/are matched correctly?
c) 2 and 3 only a) 4 only
d) 1, 2, and 3 b) 2 and 3 only
Solution- B c) 1, 2 and 3 only
Explanation d) 1 and 4 only
* The proclamation of a war emergency cannot be Answer: D
made by the President unless the Union cabinet
gives him in written that such proclamation should Explanation
be made. The 38th Amendment Act of 1975 made * Article 356 - Provisions in case of failure of
the declaration of a National Emergency immune constitutional machinery in states
from the judicial review. But this provision was
* Article 358 - Suspension of provisions of Article
subsequently deleted by the 44th Amendment Act
19 during Emergencies
of 1978.
* Article 359 -Suspension of the enforcement of the
* If the proclamation of emergency is issued at a
rights conferred by Part III during Emergencies
time when the Lok Sabha has been dissolved or
the dissolution of the Lok Sabha takes place during * Article 360 - Provisions related to Financial
the period of one month without approving the Emergency
proclamation, then the proclamation survives until
30 days from the first sitting of the Lok Sabha
after its reconstitution, provided the Rajya Sabha 91. Match the certain parts of the basic structure
has in the meantime approved it. of the constitution of India with the Supreme
Court cases in which they have been declared
as such for the first time.
90. Consider the following pairs related to Part Cases Features that are
VIII of the Constitution: part of the Basis
1. Article 356 - Provisions in case of failure Structure
of constitutional machinery in states. 1. S. R. Bommai vs Union
2. Article 358 - Suspension of enforcement of India, 1994 A. Federalism
of rights conferred by Part III during
Emergencies 2. Indra Sawhney vs

3. Article 359 - Suspension of provisions of Union of India, 1992 B. Judicial review

KPIAS (45 - D) Cell: 91332 37733


3. Kihoto Hollohan vs * A proclamation of President's Rule may be
Zachillhu and Ors, revoked by the President at any time by a
subsequent proclamation. President's rule in a
1992 C. Rule of law
State does not affect the Fundamental Rights
4. Minerva Mills vs enjoyed by the Citizens. Under the President's rule
Union of India, 1980 D. Free and fair elections ,the Parliament can delegate the power to make
laws for the state to the President or to any other
Select the correct answer using the code given
authority specified by him in this regard.
below:
a) 1-C, 2-B, 3-A, 4-D
93. Effective majority is needed in which of the
b) 1-B, 2-C, 3-D, 4-A
following cases?
c) 1-B, 2-C, 3-A, 4-D 1. Removal of Chairman of Rajya Sabha.
d) 1-A, 2-C, 3-D, 4-B 2. Removal of Speaker of Lok Sabha.
Answer: D 3. Impeachment of the President.
4. Removal of judges of the Supreme Court.
92. Consider the following statements: Select the correct answer from the codes given
1. The Parliament gets exclusive power to below.
make laws on State subject when Article a) 1 and 2 only
356 is in operation in a State.
b) 2 only
2. No Parliamentary approval is required for
revoking the President rule. c) 1, 2 and 3 only

3. President's rule in a State does not affect d) All the above


the Fundamental Rights enjoyed by the Solution:A
Citizens
* Effective Majority of the house means more than
Which of the statements given above is/are 50% of the effective strength of the house. This
correct? implies that out of the total strength, the vacant
a) 1 only seats must be deducted.

b) 2 and 3 only * Vice-President is the ex officio Chairman of Rajya


Sabha. There is no formal impeachment process
c) 1 and 3 only for the Vice-president in the Indian constitution.
d) 1, 2 and 3 He can be removed from the office by first giving
Solution : D 14 days advance notice and then a resolution
passed by a majority of all the then members of
Explanation the Rajya Sabha (effective majority) and agreed

KPIAS (46 - D) Cell: 91332 37733


to by the Lok Sabha with a simple majority. The
special majority of the Parliament with the
resolution for his removal can be introduced only
ratification by half of the total states.
in the Rajya Sabha and not in the Lok Sabha.
* Amendment bill does not require prior permission
* Speaker can be removed by the Lok Sabha by a
of the President. 24th Amendment Act 1971
resolution passed by effective majority of the
provides that when a Constitution Amendment
house as per Constitution of India [Articles 94].
Bill passed by both Houses of Parliament is
The Speaker is also removed on being disqualified
presented to the President for his assent, he
for being Lok Sabha member under sections 7 and
should give his assent. The president was made
8 of Representation of the People Act, 1951.
duty bound to give assent to a Constitution
* Removal of the Supreme Court Judges and Amendment Bill when presented to him.
Impeachment of the President, both require special
* A number of provisions in the Constitution can be
majority.
amended by a simple majority of the two Houses
of Parliament outside the scope of Article 368.
94. Consider the following statements: * Admission or establishment of new state and
1. The Article 368 provides for two types of delimitation of constituencies can be done by
amendments. Simple majority of Parliament. The bill can be
introduced either by a minister or by a private
2. The Amendment Bill does not require prior member and does not require prior permission of
permission of the President. the president.
3. Delimitation of constituencies requires
special majority of the Parliament.
95. Consider the following statements with
Which of the statements given above is/are respect to the suspension of fundamental
correct? rights during National emergency:
a) 1 and 2 only 1. Under Article 359 of Indian Constitution,
b) 2 and 3 only the Fundamental Rights guaranteed by
Articles 20 and 21 are compulsorily
c) 1 and 3 only
suspended during emergency.
d) 1, 2 and 3
2. UnlikeArticle 359,Article 358 automatically
Solution A suspends certain fundamental rights when
Explanation the emergency is proclaimed.

* Article 368 of the Indian Constitution mentions two 3. Unlike Article 359, Article 358 operates in
types of amendments to the Indian Constitution. case of both External Emergency as well
One type of amendment is by a special majority as Internal Emergency.
of the Parliament (Lok Sabha & Rajya Sabha) Which of the statements given above are correct?
and the second type of the amendment is the by a
KPIAS (47 - D) Cell: 91332 37733
a) 2 only 3. Himachal Pradesh
b) 2 and 3 only 4. Uttarakhand
c) 1 and 3 only With reference to the States mentioned above, in
terms of percentage of forest cover to the total
d) 1, 2 and 3
area of State, which one of the following is the
Answer: A correct ascending order?
* Statement 1 is incorrect. Article 358 deals with a) 2-3-1-4
the suspension of the Fundamental Rights
b) 2-1-3-4
guaranteed by Article 19, while Article 359 deals
with the suspension of other Fundamental Rights c) 3-2-4-1
(except those guaranteed by Articles 20 and 21). d) 3-2-1-4
The right to protection in respect of conviction for
offences (Article 20) and the right to life and Answer- A
personal liberty (Article 21) remain enforceable Explanation
even during emergency.
* Name of State Percentage of forest cover to total
* Statement 2 is correct. Article 358 automatically area
suspends the fundamental rights under Article 19
Chhattisgarh 41.21
as soon as the emergency is declared. On the
other hand, Article 359 only empowers the Madhya Pradesh 25.14
president to suspend the enforcement of the Himachal Pradesh 27.73
specified Fundamental Rights through a
Presidential Order. Uttarakhand 45.44

* Statement 3 is incorrect. Article 358 operates only


in case of External Emergency and not in the case 97. Consider the case of an Overseas Citizenship
of Internal Emergency. Article 359 operates in of India (OCI) Cardholder. Which of the
case of both External Emergency as well as following fundamental rights are NOT
Internal Emergency. External Emergency means available to him?
when the emergency is declared on the grounds
1. Article 15
of war or external aggression. Internal Emergency
means when the Emergency is declared on the 2. Article 16
ground of armed rebellion. 3. Article 19
4. Article 20
96. Consider the following States: Select the correct answer using the codes given
1. Chhattisgarh below:

2. Madhya Pradesh a) 1 and 2 only

KPIAS (48 - D) Cell: 91332 37733


b) 1, 2 and 3 only
neither a statutory nor a constitutional body. The
c) 3 and 4 only Law Commission is an executive body which is
d) 1, 2, 3 and 4 established by the government for a fixed tenure.

Answer: b * Statement 2 is incorrect: The Commission submits


reports to the government on various matters of a
* OCI is Overseas Citizenship of India. It is an legal nature. The reports are not binding on the
immigration status authorising a foreign citizen of government, which can either reject or accept
Indian origin to live and work in India for an them.
indefinite period. They were citizens of India on
* Statement 3 is incorrect: The Government of India
26th January 1950 or thereafter except who is or
had been a citizen of Pakistan, Bangladesh or such established the First Law Commission of
Independent India in 1955 with the then Attorney-
other country. OCI is not equivalent to Dual
General of India, Mr M. C. Setalvad.
Citizenship.
* Rights available to citizens only are - Article 15, * Objectives: Reforming the law for maximising
16, 19, 29 and 30 justice in society and promoting good governance
under the Rule of Law.

98. With reference to Law Commission of India,


consider the following: 99. Which among the following writ is also known
as "The Great writ"?
1. It is a statutory body.
a) Mandamus
2. The recommendations of law commission
b) Habeas Corpus
are binding on the government.
3. The First Law Commission of Independent c) Prohibition
India was set up in 1950. d) All the writs are known as "the great writ"
Which of the statements given above is/are Solution: B
correct? Explanation:
a) 1 and 3 only * The writ of habeas corpus-Latin for "you have
b) 2 only the body"-is known as "the Great Writ." It
c) 1 and 2 only generally, is a procedural remedy commanding a
custodian to bring a detained party, such as a
d) None of the above prisoner, before the court to show cause for the
Answer: D detainment and to prove whether the detainment
is lawful or justified.
Explanation
* Statement 1 is incorrect: The Law Commission is

KPIAS (49 - D) Cell: 91332 37733


100. With reference to linguistic reorganisation of for the creation of 16 States and 3 Union
States, consider the following statements. Territories.
1. FazlAli commission recommended that the * Andhra State (not Andhra Pradesh) was the first
reorganization of states should be on the state to be formed on a linguistic basis in India on
basis of administrative convenience rather 1 October 1953. The state of Andhra Pradesh was
than on linguistic basis. formed on November 1, 1956. Through the States
2. Andhra Pradesh was the first State to be Reorganization Act of 1956, the state of
formed on linguistic grounds in 1953. Hyderabad was split up, and its Telugu-speaking
districts (constituting Telangana) joined with the
3. Dhar commission recommended formation Andhra state.
of linguistic states.
Which of the statements given above is/are
correct?
a) Only one statement is correct
b) Two of the above statements are correct
c) All the three statements above are correct
d) None of the above statements are correct
Answer: D
Explanation:
* On 17 June 1948, Rajendra Prasad, the President
of the Constituent Assembly, set up the Linguistic
Provinces Commission (aka Dhar Commission)
to recommend whether the states should be
reorganised on linguistic basis or not. The
Commission in its report recommended that the
reorganization of states should be on the basis of
administrative convenience rather than on linguistic
basis.
* The FazlAli Commission (comprising Justice Fazal
Ali, K. M. Panikkar and H. N. Kunzru), also known
as the States Reorganization Commission, was
established in 1953. It accepted language as the
basis of reorganization of States. But it rejected
the theory of one language one State. It suggested

KPIAS (50 - D) Cell: 91332 37733


SPACE FOR ROUGH WORK

KPIAS (51 - D) Cell: 91332 37733


SPACE FOR ROUGH WORK

KPIAS (52 - D) Cell: 91332 37733

You might also like